Compute $int_{|z|=R} frac{|dz|}{|z-a|^4}$ when $R>0, |a|neq R$












0












$begingroup$


I want to compute $int_{|z|=R} frac{|dz|}{|z-a|^4}$ when $R>0, |a|neq R$.
I did so by parametrizing the circle, but this way required many pages of tedious calculations.



Do you have a more straightforward way to do it?



Let me mention that my issue here is that we integrate by $|dz|$ not by $dz$, so all the known theorems I know fail to be applied -at least in this form of the integral.



Thanks










share|cite|improve this question











$endgroup$

















    0












    $begingroup$


    I want to compute $int_{|z|=R} frac{|dz|}{|z-a|^4}$ when $R>0, |a|neq R$.
    I did so by parametrizing the circle, but this way required many pages of tedious calculations.



    Do you have a more straightforward way to do it?



    Let me mention that my issue here is that we integrate by $|dz|$ not by $dz$, so all the known theorems I know fail to be applied -at least in this form of the integral.



    Thanks










    share|cite|improve this question











    $endgroup$















      0












      0








      0





      $begingroup$


      I want to compute $int_{|z|=R} frac{|dz|}{|z-a|^4}$ when $R>0, |a|neq R$.
      I did so by parametrizing the circle, but this way required many pages of tedious calculations.



      Do you have a more straightforward way to do it?



      Let me mention that my issue here is that we integrate by $|dz|$ not by $dz$, so all the known theorems I know fail to be applied -at least in this form of the integral.



      Thanks










      share|cite|improve this question











      $endgroup$




      I want to compute $int_{|z|=R} frac{|dz|}{|z-a|^4}$ when $R>0, |a|neq R$.
      I did so by parametrizing the circle, but this way required many pages of tedious calculations.



      Do you have a more straightforward way to do it?



      Let me mention that my issue here is that we integrate by $|dz|$ not by $dz$, so all the known theorems I know fail to be applied -at least in this form of the integral.



      Thanks







      complex-integration






      share|cite|improve this question















      share|cite|improve this question













      share|cite|improve this question




      share|cite|improve this question








      edited Oct 24 '17 at 5:28







      perlman

















      asked Oct 24 '17 at 5:22









      perlmanperlman

      8110




      8110






















          2 Answers
          2






          active

          oldest

          votes


















          0












          $begingroup$

          Try This



          $f^n(z)$ = $frac {n!}{2(pi)n}$ $int$ $frac {f(w) dw}{(w-z)^{n+1}}$






          share|cite|improve this answer











          $endgroup$













          • $begingroup$
            Can you make it a bit more clear? Specifically, how do I treat the $|dz|$?
            $endgroup$
            – perlman
            Oct 24 '17 at 5:47










          • $begingroup$
            I just assumed the absolute value sign was meant to be around the function. I don't think I've specifically dealt with a case you're talking about, because I tried googling it, and |dz| turns out to be some kind of arc length.
            $endgroup$
            – Math Model
            Oct 24 '17 at 5:57



















          0












          $begingroup$

          After parametrising $z= R e^{itheta}$ and after a lot of simplifications you will get
          $$I = int_{0}^{2pi} dtheta frac{R}{[(Rcostheta-Re(a))^2+(Rsintheta-Im(a))^2]^2_{}}.$$
          To proceed further : go back to complex variable $Z = e^{itheta}$, rewrite it in terms of $costheta=frac{1}{2}(z+frac{1}{z})$ and $sintheta=frac{1}{2i}(z-frac{1}{z})$. The resulting integral can be evaluated using complex integration methods.






          share|cite|improve this answer









          $endgroup$














            Your Answer





            StackExchange.ifUsing("editor", function () {
            return StackExchange.using("mathjaxEditing", function () {
            StackExchange.MarkdownEditor.creationCallbacks.add(function (editor, postfix) {
            StackExchange.mathjaxEditing.prepareWmdForMathJax(editor, postfix, [["$", "$"], ["\\(","\\)"]]);
            });
            });
            }, "mathjax-editing");

            StackExchange.ready(function() {
            var channelOptions = {
            tags: "".split(" "),
            id: "69"
            };
            initTagRenderer("".split(" "), "".split(" "), channelOptions);

            StackExchange.using("externalEditor", function() {
            // Have to fire editor after snippets, if snippets enabled
            if (StackExchange.settings.snippets.snippetsEnabled) {
            StackExchange.using("snippets", function() {
            createEditor();
            });
            }
            else {
            createEditor();
            }
            });

            function createEditor() {
            StackExchange.prepareEditor({
            heartbeatType: 'answer',
            autoActivateHeartbeat: false,
            convertImagesToLinks: true,
            noModals: true,
            showLowRepImageUploadWarning: true,
            reputationToPostImages: 10,
            bindNavPrevention: true,
            postfix: "",
            imageUploader: {
            brandingHtml: "Powered by u003ca class="icon-imgur-white" href="https://imgur.com/"u003eu003c/au003e",
            contentPolicyHtml: "User contributions licensed under u003ca href="https://creativecommons.org/licenses/by-sa/3.0/"u003ecc by-sa 3.0 with attribution requiredu003c/au003e u003ca href="https://stackoverflow.com/legal/content-policy"u003e(content policy)u003c/au003e",
            allowUrls: true
            },
            noCode: true, onDemand: true,
            discardSelector: ".discard-answer"
            ,immediatelyShowMarkdownHelp:true
            });


            }
            });














            draft saved

            draft discarded


















            StackExchange.ready(
            function () {
            StackExchange.openid.initPostLogin('.new-post-login', 'https%3a%2f%2fmath.stackexchange.com%2fquestions%2f2487046%2fcompute-int-z-r-fracdzz-a4-when-r0-a-neq-r%23new-answer', 'question_page');
            }
            );

            Post as a guest















            Required, but never shown

























            2 Answers
            2






            active

            oldest

            votes








            2 Answers
            2






            active

            oldest

            votes









            active

            oldest

            votes






            active

            oldest

            votes









            0












            $begingroup$

            Try This



            $f^n(z)$ = $frac {n!}{2(pi)n}$ $int$ $frac {f(w) dw}{(w-z)^{n+1}}$






            share|cite|improve this answer











            $endgroup$













            • $begingroup$
              Can you make it a bit more clear? Specifically, how do I treat the $|dz|$?
              $endgroup$
              – perlman
              Oct 24 '17 at 5:47










            • $begingroup$
              I just assumed the absolute value sign was meant to be around the function. I don't think I've specifically dealt with a case you're talking about, because I tried googling it, and |dz| turns out to be some kind of arc length.
              $endgroup$
              – Math Model
              Oct 24 '17 at 5:57
















            0












            $begingroup$

            Try This



            $f^n(z)$ = $frac {n!}{2(pi)n}$ $int$ $frac {f(w) dw}{(w-z)^{n+1}}$






            share|cite|improve this answer











            $endgroup$













            • $begingroup$
              Can you make it a bit more clear? Specifically, how do I treat the $|dz|$?
              $endgroup$
              – perlman
              Oct 24 '17 at 5:47










            • $begingroup$
              I just assumed the absolute value sign was meant to be around the function. I don't think I've specifically dealt with a case you're talking about, because I tried googling it, and |dz| turns out to be some kind of arc length.
              $endgroup$
              – Math Model
              Oct 24 '17 at 5:57














            0












            0








            0





            $begingroup$

            Try This



            $f^n(z)$ = $frac {n!}{2(pi)n}$ $int$ $frac {f(w) dw}{(w-z)^{n+1}}$






            share|cite|improve this answer











            $endgroup$



            Try This



            $f^n(z)$ = $frac {n!}{2(pi)n}$ $int$ $frac {f(w) dw}{(w-z)^{n+1}}$







            share|cite|improve this answer














            share|cite|improve this answer



            share|cite|improve this answer








            edited Oct 24 '17 at 5:55

























            answered Oct 24 '17 at 5:41









            Math ModelMath Model

            445




            445












            • $begingroup$
              Can you make it a bit more clear? Specifically, how do I treat the $|dz|$?
              $endgroup$
              – perlman
              Oct 24 '17 at 5:47










            • $begingroup$
              I just assumed the absolute value sign was meant to be around the function. I don't think I've specifically dealt with a case you're talking about, because I tried googling it, and |dz| turns out to be some kind of arc length.
              $endgroup$
              – Math Model
              Oct 24 '17 at 5:57


















            • $begingroup$
              Can you make it a bit more clear? Specifically, how do I treat the $|dz|$?
              $endgroup$
              – perlman
              Oct 24 '17 at 5:47










            • $begingroup$
              I just assumed the absolute value sign was meant to be around the function. I don't think I've specifically dealt with a case you're talking about, because I tried googling it, and |dz| turns out to be some kind of arc length.
              $endgroup$
              – Math Model
              Oct 24 '17 at 5:57
















            $begingroup$
            Can you make it a bit more clear? Specifically, how do I treat the $|dz|$?
            $endgroup$
            – perlman
            Oct 24 '17 at 5:47




            $begingroup$
            Can you make it a bit more clear? Specifically, how do I treat the $|dz|$?
            $endgroup$
            – perlman
            Oct 24 '17 at 5:47












            $begingroup$
            I just assumed the absolute value sign was meant to be around the function. I don't think I've specifically dealt with a case you're talking about, because I tried googling it, and |dz| turns out to be some kind of arc length.
            $endgroup$
            – Math Model
            Oct 24 '17 at 5:57




            $begingroup$
            I just assumed the absolute value sign was meant to be around the function. I don't think I've specifically dealt with a case you're talking about, because I tried googling it, and |dz| turns out to be some kind of arc length.
            $endgroup$
            – Math Model
            Oct 24 '17 at 5:57











            0












            $begingroup$

            After parametrising $z= R e^{itheta}$ and after a lot of simplifications you will get
            $$I = int_{0}^{2pi} dtheta frac{R}{[(Rcostheta-Re(a))^2+(Rsintheta-Im(a))^2]^2_{}}.$$
            To proceed further : go back to complex variable $Z = e^{itheta}$, rewrite it in terms of $costheta=frac{1}{2}(z+frac{1}{z})$ and $sintheta=frac{1}{2i}(z-frac{1}{z})$. The resulting integral can be evaluated using complex integration methods.






            share|cite|improve this answer









            $endgroup$


















              0












              $begingroup$

              After parametrising $z= R e^{itheta}$ and after a lot of simplifications you will get
              $$I = int_{0}^{2pi} dtheta frac{R}{[(Rcostheta-Re(a))^2+(Rsintheta-Im(a))^2]^2_{}}.$$
              To proceed further : go back to complex variable $Z = e^{itheta}$, rewrite it in terms of $costheta=frac{1}{2}(z+frac{1}{z})$ and $sintheta=frac{1}{2i}(z-frac{1}{z})$. The resulting integral can be evaluated using complex integration methods.






              share|cite|improve this answer









              $endgroup$
















                0












                0








                0





                $begingroup$

                After parametrising $z= R e^{itheta}$ and after a lot of simplifications you will get
                $$I = int_{0}^{2pi} dtheta frac{R}{[(Rcostheta-Re(a))^2+(Rsintheta-Im(a))^2]^2_{}}.$$
                To proceed further : go back to complex variable $Z = e^{itheta}$, rewrite it in terms of $costheta=frac{1}{2}(z+frac{1}{z})$ and $sintheta=frac{1}{2i}(z-frac{1}{z})$. The resulting integral can be evaluated using complex integration methods.






                share|cite|improve this answer









                $endgroup$



                After parametrising $z= R e^{itheta}$ and after a lot of simplifications you will get
                $$I = int_{0}^{2pi} dtheta frac{R}{[(Rcostheta-Re(a))^2+(Rsintheta-Im(a))^2]^2_{}}.$$
                To proceed further : go back to complex variable $Z = e^{itheta}$, rewrite it in terms of $costheta=frac{1}{2}(z+frac{1}{z})$ and $sintheta=frac{1}{2i}(z-frac{1}{z})$. The resulting integral can be evaluated using complex integration methods.







                share|cite|improve this answer












                share|cite|improve this answer



                share|cite|improve this answer










                answered Oct 24 '17 at 6:34









                SunyamSunyam

                167214




                167214






























                    draft saved

                    draft discarded




















































                    Thanks for contributing an answer to Mathematics Stack Exchange!


                    • Please be sure to answer the question. Provide details and share your research!

                    But avoid



                    • Asking for help, clarification, or responding to other answers.

                    • Making statements based on opinion; back them up with references or personal experience.


                    Use MathJax to format equations. MathJax reference.


                    To learn more, see our tips on writing great answers.




                    draft saved


                    draft discarded














                    StackExchange.ready(
                    function () {
                    StackExchange.openid.initPostLogin('.new-post-login', 'https%3a%2f%2fmath.stackexchange.com%2fquestions%2f2487046%2fcompute-int-z-r-fracdzz-a4-when-r0-a-neq-r%23new-answer', 'question_page');
                    }
                    );

                    Post as a guest















                    Required, but never shown





















































                    Required, but never shown














                    Required, but never shown












                    Required, but never shown







                    Required, but never shown

































                    Required, but never shown














                    Required, but never shown












                    Required, but never shown







                    Required, but never shown







                    Popular posts from this blog

                    Biblatex bibliography style without URLs when DOI exists (in Overleaf with Zotero bibliography)

                    ComboBox Display Member on multiple fields

                    Is it possible to collect Nectar points via Trainline?